Is every subset of a totally bounded set also totally bounded?

Click For Summary
Not every subset of a totally bounded set is totally bounded. A totally bounded set F allows for the construction of a finite subset F_0 that covers F with open balls of radius ε. However, this does not guarantee that a subset E of F will also be totally bounded, as F_0 may not intersect E. An example illustrates this point, where E could be a fractal within an open ball F. The discussion concludes that while subsets may not be totally bounded, a method exists to demonstrate that a totally bounded subset of a totally bounded set is indeed totally bounded.
Fredrik
Staff Emeritus
Science Advisor
Homework Helper
Insights Author
Gold Member
Messages
10,876
Reaction score
423
Not really homework, but a textbook-style question...

Homework Statement


Is every subset of a totally bounded set (of a metric space) totally bounded?


Homework Equations



F is said to be totally bounded if, for every \epsilon>0, there's a finite subset F_0\subset F such that F\subset\bigcup_{x\in F_0}B(x,\epsilon), where B(x,\epsilon) is the open ball of radius \epsilon around x.

The Attempt at a Solution


Suppose that E\subset F, and that F is totally bounded. Let \epsilon>0 be arbitrary. We know that there exists a finite set F_0\subset F such that E\subset F\subset\bigcup_{x\in F_0}B(x,\epsilon), but this doesn't seem to help, since F_0 doesn't have to be a subset of E. We might even have F_0\cap E=\emptyset. So now I'm starting to think that maybe E doesn't have to be totally bounded at all. For example, if F is some open ball in \mathbb R^2 and E is some kind of fractal or something.
 
Physics news on Phys.org
No, a totally bounded subset of a totally bounded set is totally bounded. Most definitions are a bit looser than yours. But you can still prove it. Pick a finite set F0 to cover with balls of radius e/2. Can you use that to construct a finite set E0 that covers E with radius e?
 
Ah, I get it now. I'll just pick one point from each non-empty B(x,\epsilon/2)\cap E with x\in F_0, and take those points to be my E0. Then I consider open balls around those points, and I need to take these balls to have twice the radius to ensure that they cover E (by covering the old balls that had non-empty intersection with E). Thanks.
 
Question: A clock's minute hand has length 4 and its hour hand has length 3. What is the distance between the tips at the moment when it is increasing most rapidly?(Putnam Exam Question) Answer: Making assumption that both the hands moves at constant angular velocities, the answer is ## \sqrt{7} .## But don't you think this assumption is somewhat doubtful and wrong?

Similar threads

Replies
1
Views
2K
  • · Replies 4 ·
Replies
4
Views
1K
  • · Replies 16 ·
Replies
16
Views
2K
Replies
1
Views
3K
  • · Replies 1 ·
Replies
1
Views
2K
  • · Replies 1 ·
Replies
1
Views
2K
  • · Replies 5 ·
Replies
5
Views
2K
  • · Replies 4 ·
Replies
4
Views
2K
  • · Replies 5 ·
Replies
5
Views
2K
  • · Replies 2 ·
Replies
2
Views
2K